Difference between revisions of "1961 AHSME Problems/Problem 37"

m (Problem)
m (Problem)
Line 4: Line 4:
 
and <math>A</math> can beat <math>C</math> by <math>28</math> yards. Then <math>d</math>, in yards, equals:
 
and <math>A</math> can beat <math>C</math> by <math>28</math> yards. Then <math>d</math>, in yards, equals:
  
<math>\textbf{(A)}\ \text{Not determined by the given information} \qquad </math><math>\textbf{(B)}\ 58\qquad
+
<math>\textbf{(A)}\ \text{Not determined by the given information} \qquad  
 +
\textbf{(B)}\ 58\qquad
 
\textbf{(C)}\ 100\qquad
 
\textbf{(C)}\ 100\qquad
 
\textbf{(D)}\ 116\qquad
 
\textbf{(D)}\ 116\qquad

Revision as of 20:29, 31 May 2018

Problem

In racing over a distance $d$ at uniform speed, $A$ can beat $B$ by $20$ yards, $B$ can beat $C$ by $10$ yards, and $A$ can beat $C$ by $28$ yards. Then $d$, in yards, equals:

$\textbf{(A)}\ \text{Not determined by the given information} \qquad  \textbf{(B)}\ 58\qquad \textbf{(C)}\ 100\qquad \textbf{(D)}\ 116\qquad \textbf{(E)}\ 120$

Solution

Let $a$ be speed of $A$, $b$ be speed of $B$, and $c$ be speed of $C$.

Person $A$ finished the track in $\frac{d}{a}$ minutes, so $B$ traveled $\frac{db}{a}$ yards at the same time. Since $B$ is $20$ yards from the finish line, the first equation is \[\frac{db}{a} + 20 = d\] Using similar steps, the second and third equation are, respectively, \[\frac{dc}{b} + 10 = d\] \[\frac{dc}{a} + 28 = d\] Get rid of the denominator in all three equations by multiplying both sides by the denominator in each equation. \[db + 20a = da\] \[dc + 10b = db\] \[dc + 28a = da\] These equations can be rearranged to get the following. \[(d-20)a = db\] \[(d-10)b = dc\] \[(d-28)a = dc\]

Solving for $b$ in the first equation yields $\frac{(d-20)a}{d} = b$. Substituting for $b$ and solving for $c$ in the second equation yields $\frac{(d-10)(d-20)a}{d^2} = c$. Substituting for $c$ in the third equation yields \[(d-28)a = \frac{(d-10)(d-20)a}{d}\] \[d^2 - 28d = d^2 - 30d + 200\] Solve the equation to get $d = 100$. Thus, the track is $100$ yards long, which is answer choice $\boxed{\textbf{(C)}}$.

See Also

1961 AHSC (ProblemsAnswer KeyResources)
Preceded by
Problem 36
Followed by
Problem 38
1 2 3 4 5 6 7 8 9 10 11 12 13 14 15 16 17 18 19 20 21 22 23 24 25 26 27 28 29 30 31 32 33 34 35 36 37 38 39 40
All AHSME Problems and Solutions

The problems on this page are copyrighted by the Mathematical Association of America's American Mathematics Competitions. AMC logo.png